Những câu hỏi liên quan
Lê Minh Đức
Xem chi tiết
Akai Haruma
9 tháng 10 2017 lúc 17:28

Ta có \(a^2b^2+b^2c^2+c^2a^2\geq a^2b^2c^2\Leftrightarrow \frac{1}{a^2}+\frac{1}{b^2}+\frac{1}{c^2}\geq 1\)

BĐT cần chứng minh tương đương với \(\frac{\frac{1}{c^3}}{\frac{1}{a^2}+\frac{1}{b^2}}+\frac{\frac{1}{b^3}}{\frac{1}{a^2}+\frac{1}{c^2}}+\frac{\frac{1}{a^3}}{\frac{1}{b^2}+\frac{1}{c^2}}\geq \frac{\sqrt{3}}{2}\)

Đặt \((\frac{1}{a},\frac{1}{b},\frac{1}{c})=(x,y,z)\). Bài toán trở thành: 

Cho \(x,y,z>0|x^2+y^2+z^2\geq 1\). CMR \(P=\frac{x^3}{y^2+z^2}+\frac{y^3}{z^2+x^2}+\frac{z^3}{x^2+y^2}\geq \frac{\sqrt{3}}{2}\)

Lời giải:

 Áp dụng BĐT Cauchy -Schwarz:

\(P=\frac{x^4}{xy^2+xz^2}+\frac{y^4}{yz^2+yx^2}+\frac{z^4}{zx^2+zy^2}\geq \frac{(x^2+y^2+^2)^2}{x^2(y+z)+y^2(x+z)+z^2(x+y)}\) (1)

Không mất tính tổng quát, giả sử \(x\geq y\geq z\Rightarrow x^2\geq y^2\geq z^2\) 

Và \(y+z\leq z+x\leq x+y\). Khi đó, áp dụng BĐT Chebyshev: 

\(3[x^2(y+z)+y^2(x+z)+z^2(x+y)]\leq (x^2+y^2+z^2)(y+z+x+z+x+y)\)

\(\Leftrightarrow x^2(y+z)+y^2(x+z)+z^2(x+y)\leq \frac{2(x^2+y^2+z^2)(x+y+z)}{3}\)

Theo hệ quả của BĐT Am-Gm thì: \((x+y+z)^2\leq 3(x^2+y^2+z^2)\Rightarrow x+y+z\leq \sqrt{3(x^2+y^2+z^2)}\)

\(\Rightarrow x^2(y+z)+y^2(x+z)+z^2(x+y)\leq \frac{2(x^2+y^2+z^2)\sqrt{3(x^2+y^2+z^2)}}{3}\) (2)

Từ (1),(2) suy ra \(P\geq \frac{3(x^2+y^2+z^2)^2}{2(x^2+y^2+z^2)\sqrt{3(x^2+y^2+z^2)}}=\frac{\sqrt{3(x^2+y^2+z^2)}}{2}\geq \frac{\sqrt{3}}{2}\)

Ta có đpcm

Dáu bằng xảy ra khi \(x=y=z=\frac{1}{\sqrt{3}}\Leftrightarrow a=b=c=\sqrt{3}\)

Kiệt Nguyễn
5 tháng 5 2020 lúc 12:58

Đặt \(x=\frac{1}{a};y=\frac{1}{b};z=\frac{1}{c}\)

Khi đó giả thiết được viết lại là \(x^2+y^2+z^2\ge1\)và ta cần chứng minh \(\frac{x^3}{y^2+z^2}+\frac{y^3}{z^2+x^2}+\frac{z^3}{x^2+y^2}\ge\frac{\sqrt{3}}{2}\)(*)

Áp dụng BĐT Bunhiacopxki dạng phân thức, ta được:

\(VT_{\left(^∗\right)}=\frac{x^4}{x\left(y^2+z^2\right)}+\frac{y^4}{y\left(z^2+x^2\right)}+\frac{z^4}{z\left(x^2+y^2\right)}\)\(\ge\frac{\left(x^2+y^2+z^2\right)^2}{x\left(y^2+z^2\right)+y\left(z^2+x^2\right)+z\left(x^2+y^2\right)}\)

Đến đây ta đi chứng minh \(\frac{\left(x^2+y^2+z^2\right)^2}{x\left(y^2+z^2\right)+y\left(z^2+x^2\right)+z\left(x^2+y^2\right)}\ge\frac{\sqrt{3}}{2}\)

\(\Leftrightarrow2\left(x^2+y^2+z^2\right)^2\)\(\ge\sqrt{3}\left[x\left(y^2+z^2\right)+y\left(z^2+x^2\right)+z\left(x^2+y^2\right)\right]\)

Ta có: \(x\left(y^2+z^2\right)=\frac{1}{\sqrt{2}}\sqrt{2x^2\left(y^2+z^2\right)\left(y^2+z^2\right)}\)\(\le\frac{1}{\sqrt{2}}\sqrt{\left(\frac{2x^2+y^2+z^2+y^2+z^2}{3}\right)^3}\)

\(=\frac{2\sqrt{3}}{9}\left(x^2+y^2+z^2\right)\sqrt{x^2+y^2+z^2}\)

Tương tự ta có: \(y\left(z^2+x^2\right)\le\frac{2\sqrt{3}}{9}\left(x^2+y^2+z^2\right)\sqrt{x^2+y^2+z^2}\)

\(z\left(x^2+y^2\right)\le\frac{2\sqrt{3}}{9}\left(x^2+y^2+z^2\right)\sqrt{x^2+y^2+z^2}\)

Cộng theo vế của 3 BĐT trên, ta được: 

\(\text{∑}_{cyc}\left[x\left(y^2+z^2\right)\right]\le\frac{2\sqrt{3}}{3}\left(x^2+y^2+z^2\right)\sqrt{x^2+y^2+z^2}\)

\(\Leftrightarrow\sqrt{3}\text{∑}_{cyc}\left[x\left(y^2+z^2\right)\right]\le2\left(x^2+y^2+z^2\right)\sqrt{x^2+y^2+z^2}\)

Cuối cùng ta cần chứng minh được

\(2\left(x^2+y^2+z^2\right)\sqrt{x^2+y^2+z^2}\le2\left(x^2+y^2+z^2\right)^2\)

\(\Leftrightarrow x^2+y^2+z^2\ge1\)(đúng)

Đẳng thức xảy ra khi \(x=y=z=\frac{1}{\sqrt{3}}\Rightarrow a=b=c=\sqrt{3}\)

Khách vãng lai đã xóa
Chàng trai bóng đêm
Xem chi tiết
Pham Quoc Cuong
4 tháng 9 2018 lúc 17:33

\(\frac{2a^2}{a+b^2}=2a-\frac{2ab^2}{a+b^2}\ge2a-\frac{2ab^2}{2b\sqrt{a}}=2a-b\sqrt{a}\ge2a-\frac{b+ba}{2}\) 

Tương tự rồi cộng từng vế ta có: 

\(\frac{2a^2}{a+b^2}+\frac{2b^2}{b+c^2}+\frac{2c^2}{c+a^2}\ge\frac{3}{2}\left(a+b+c\right)-\frac{ab+bc+ca}{2}\) 

Lại có: \(\left(a+b+c\right)^2\left(a^2+b^2+c^2\right)\ge3\left(ab+bc+ca\right)^2\Rightarrow a+b+c\ge ab+bc+ca\) 

\(\Rightarrow VT\ge\frac{3}{2}\left(a+b+c\right)-\frac{a+b+c}{2}\ge a+b+c\) 

Dấu "=' khi a=b=c=1

zZz Cool Kid_new zZz
11 tháng 6 2020 lúc 10:55

Làm 2 cách nhá 

\(\frac{2a^2}{a+b^2}=\frac{2a^2}{\frac{a^2+1}{2}+b^2}=\frac{4a^2}{a^2+2b^2+1}=\frac{4a^4}{a^4+2a^2b^2+a^2}\)

Tương tự rồi theo Cauchy Schwarz ta có được:

\(LHS\ge\frac{\left(2a^2+2b^2+2c^2\right)^2}{a^4+b^4+c^4+2a^2b^2+2b^2c^2+2c^2a^2+3}=\frac{36}{\left(a^2+b^2+c^2\right)^2+3}=\frac{36}{12}=3\)

Đẳng thức xảy ra tại a=b=c=1

Khách vãng lai đã xóa
zZz Cool Kid_new zZz
11 tháng 6 2020 lúc 11:02

Theo BĐT Cauchy Schwarz ta dễ có:

\(LHS=\frac{4a^4}{2a^3+2a^2b^2}+\frac{4b^4}{2b^3+2b^2c^2}+\frac{4c^4}{2c^3+2c^2a^2}\)

\(\ge\frac{\left(2a^2+2b^2+2c^2\right)^2}{2a^3+2b^3+2c^3+2a^2b^2+2b^2c^2+2c^2a^2}\)

\(\ge\frac{36}{\left(a^4+a^2\right)+\left(b^4+b^2\right)+\left(c^4+c^2\right)+2a^2b^2+2b^2c^2+2c^2a^2}\)

\(=\frac{36}{\left(a^2+b^2+c^2\right)^2+a^2+b^2+c^2}\)

\(=3\)

Mà \(3=a^2+b^2+c^2\ge\frac{1}{3}\left(a+b+c\right)^2\Rightarrow a+b+c\le3\Rightarrowđpcm\)

Khách vãng lai đã xóa
thánh yasuo lmht
Xem chi tiết
Thắng Nguyễn
8 tháng 2 2017 lúc 13:26

\(BDT\LeftrightarrowΣ\frac{a^2}{a+b^2}\ge\frac{a+b+c}{2}\)

Áp dụng BDT C-S dạng Engel ta có:

\(Σ\frac{a^2}{a+b^2}=\text{ }Σ\frac{a^4}{a^3+a^2b^2}\ge\frac{\left(a^2+b^2+c^2\right)^2}{Σa^3+a^2b^2}\)

Vậy đi chứng minh \(\frac{\left(a^2+b^2+c^2\right)^2}{Σa^3+a^2b^2}\ge\frac{a+b+c}{2}\)

Hay \(2\left(a^2+b^2+c^2\right)^2\ge\left(a+b+c\right)Σ\left(a^3+b^2c^2\right)\)

\(\hept{\begin{cases}a+b+c=3u\\ab+ac+bc=3v^2\\abc=w^3\end{cases}}\)

Bởi vì điều kiện không phụ thuộc vào \(w^3\), ta thấy rằng bất đẳng thức cuối cùng là một bất đẳng thức tuyến tính của \(w^3\), đủ để chứng minh rằng bất đẳng thức cuối cùng đạt một giá trị cực đại là \(w^3\), xảy ra trong trường hợp hai biến bằng nhau hoặc có thể cho \(w^3\rightarrow0^+\)

Sau khi biến đổi đồng nhất ta cần chứng minh.

 

\(\left(2\left(a^2+b^2+c^2\right)^2-\left(a+b+c\right)\left(a^3+b^3+c^3\right)\right)^2\left(a^2+b^2+c^2\right)\)

\(\ge3\left(a+b+c\right)^2\left(a^2b^2+a^2c^2+b^2c^2\right)^2\)

*)\(b=c=1\) Ta được

\(\left(a-1\right)^2\left(a^8-2a^7+17a^6-8a^5+75a^4-10a^3+73a^2-4a+20\right)\ge0\) ( hiển nhiên đúng)

*)\(w^3\rightarrow0^+\) để  \(c\rightarrow0^+\) và \(b=1\), ta đc:

\(a^{10}-2a^9+10a^8-12a^7+26a^6-26a^5+26a^4-12a^3+10a^2-2a+1\ge0\)( cũng đúng)

Thắng Nguyễn
8 tháng 2 2017 lúc 17:58

cách này phiêu quá lát mk làm lại

Thắng Nguyễn
8 tháng 2 2017 lúc 20:21

Khi \(a+b+c\le\sqrt{3\left(a^2+b^2+c^2\right)}=3\), viết lại BĐT cần chứng minh

\(\frac{a^2}{a+b^2}+\frac{b^2}{b+c^2}+\frac{c^2}{c+a^2}\ge\frac{3}{2}\)

Đặt \(\hept{\begin{cases}a^2=x\\b^2=y\\c^2=z\end{cases}}\) ta có \(x+y+z=3\). Áp dụng AM-GM và Cauchy-Schwarz ta có:

\(Σ\frac{a^2}{a+b^2}=Σ\frac{x}{\sqrt{x}+y}=Σ\frac{x}{\sqrt{\frac{x\left(x+y+z\right)}{3}+y}}\)

\(=Σ\frac{6x}{2\sqrt{3x\left(x+y+z\right)}+6y}\geΣ\frac{6x}{3x+x+y+z+6y}=Σ\frac{6x}{4x+7y+z}\)

\(=Σ\frac{6x^2}{4x^2+7xy+xz}\ge\frac{6\left(x+y+z\right)^2}{Σ\left(4x^2+7xy+xz\right)}=\frac{3}{2}\) 

Hoàn thành!

Đỗ Đức Đạt
Xem chi tiết
l҉o҉n҉g҉ d҉z҉
6 tháng 4 2021 lúc 13:56

Ta có : \(\frac{a}{b^2c^2}+\frac{b}{c^2a^2}+\frac{c}{a^2b^2}=\frac{a^4}{a^3b^2c^2}+\frac{b^4}{b^3c^2a^2}+\frac{c^4}{c^3a^2b^2}\)

Áp dụng bất đẳng thức Cauchy-Schwarz dạng Engel và giả thiết a2 + b2 + c2 = 3abc ta có :

\(\frac{a^4}{a^3b^2c^2}+\frac{b^4}{b^3c^2a^2}+\frac{c^4}{c^3a^2b^2}\ge\frac{\left(a^2+b^2+c^2\right)^2}{a^2b^2c^2\left(a+b+c\right)}=\frac{\left(3abc\right)^2}{a^2b^2c^2\left(a+b+c\right)}=\frac{9}{a+b+c}\left(đpcm\right)\)

Đẳng thức xảy ra <=> a=b=c=1

Khách vãng lai đã xóa
pham ngoc huyen tram
Xem chi tiết
Trần Công Văn
14 tháng 4 2020 lúc 19:57

1+1.2=?

Khách vãng lai đã xóa
Thanh Tùng DZ
14 tháng 4 2020 lúc 20:09

cho mình hỏi đề đúng không vậy

Khách vãng lai đã xóa
Agami Raito
Xem chi tiết
Nguyễn Thành Trương
18 tháng 7 2019 lúc 20:46

Áp dụng bđt cô-si, ta có: \(a+b^2\le\dfrac{a^2+1}{2}+b^2=\dfrac{a^2+2b^2+1}{2}\)

=>\(\dfrac{2a^2}{a+b^2}\ge\dfrac{4a^2}{a^2+2b^2+1}\)

CMTT: Khi đó: \(\dfrac{2a^2}{a+b^2}+\dfrac{2b^2}{b+c^2}+\dfrac{2c^2}{c+a^2}\ge\dfrac{4a^2}{a^2+2b^2+1}+\dfrac{4b^2}{b^2+2c^2+1}+\dfrac{4c^2}{c^2+2a^2+1}\)

Áp dụng bđt Sơ-vác, ta có:

\(\dfrac{4a^4}{a^4+2a^2b^2+a^2}+\dfrac{4b^4}{b^4+2b^2c^2+b^2}+\dfrac{4c^4}{c^4+2c^2a^2+c^2}\ge\dfrac{4\left(a^2+b^2+c^2\right)^2}{\left(a^2+b^2+c^2\right)^2+a^2+b^2+c^2}=\dfrac{4.3^2}{3^2+3}=3\)

Do đó: \(\dfrac{2a^2}{a+b^2}+\dfrac{2b^2}{b+c^2}+\dfrac{2c^2}{c+a^2}\ge\dfrac{4a^2}{a^2+2b^2+1}+\dfrac{4b^2}{b^2+2c^2+1}+\dfrac{4c^2}{c^2+2a^2+1}\ge3\)

Vì \(a+b+c\le\sqrt{3\left(a^2+b^2+c^2\right)}=3\)

=>\(\dfrac{2a^2}{a+b^2}+\dfrac{2b^2}{b+c^2}+\dfrac{2c^2}{c+a^2}\ge a+b+c\)

Dấu "=" xảy ra khi a=b=c=1

=>ĐPCM

Phùng Gia Bảo
Xem chi tiết
Darlingg🥝
15 tháng 1 2020 lúc 12:50

Áp dụng BĐT Bunhiacopxky ta có:

\(\left(a^2+2c^2\right)\left(1+2\right)\ge\left(a+2c^2\right)\)

\(\Rightarrow\sqrt{a^2+2c^2}\ge\frac{a+2c}{3}\)

\(\Rightarrow\frac{\sqrt{a^2+2c^2}}{ac}\ge\frac{a+2c}{\sqrt{3ac}}=\frac{ab+2bc}{\sqrt{3abc}}\)

\(\Rightarrow\hept{\begin{cases}\frac{\sqrt{c^2+2b^2}}{bc}\ge\frac{ac+2ab}{\sqrt{3abc}}\\\frac{\sqrt{b^2+2a^2}}{ab}\ge\frac{bc+2ac}{\sqrt{abc}}\end{cases}}\)

Ta được BĐT:

\(VT\ge\frac{1}{3}.\frac{ab+2abc+ac+2ab+bc+2ac}{abc}=\frac{1}{3}.\frac{3\left(ab+bc+ac\right)}{abc}\)

\(=\frac{1}{\sqrt{3}}.\frac{3abc}{abc}=3\)

=> đpcm

P/S: Làm tắt vs đoạn này k^o chắc mấy :V

Khách vãng lai đã xóa
Nyatmax
15 tháng 1 2020 lúc 13:48

Repair đề \(\Sigma_{cyc}\frac{\sqrt{2a^2+b^2}}{ab}\ge3\sqrt{3}\).Because dấu '=' xảy ra khi \(a=b=c=3\)

Không use condition của đề bài :))

Ta co:

\(VT=\sqrt{\frac{a}{b}+\frac{a}{b}+\frac{b}{a}}+\sqrt{\frac{b}{c}+\frac{b}{c}+\frac{c}{b}}+\sqrt{\frac{c}{a}+\frac{c}{a}+\frac{a}{c}}\)

\(\Rightarrow VT\ge\sqrt{3\sqrt[3]{\frac{a}{b}}}+\sqrt{3\sqrt[3]{\frac{b}{c}}}+\sqrt{3\sqrt[3]{\frac{c}{a}}}\ge3\sqrt[3]{\sqrt{3\sqrt[3]{\frac{a}{b}}.\sqrt{3\sqrt[3]{\frac{b}{c}}.\sqrt{3\sqrt[3]{\frac{c}{a}}}}}}=3\sqrt{3}\)

equelity iff \(a=b=c=3\)

Khách vãng lai đã xóa
Phùng Minh Quân
15 tháng 1 2020 lúc 15:02

\(ab+bc+ca=abc\)\(\Leftrightarrow\)\(\frac{1}{a}+\frac{1}{b}+\frac{1}{c}=1\)

\(\Sigma\frac{\sqrt{2a^2+b^2}}{ab}\ge\Sigma\frac{\sqrt{\frac{\left(2a+b\right)^2}{3}}}{ab}=\frac{1}{\sqrt{3}}\Sigma\frac{2a+b}{ab}=\frac{1}{\sqrt{3}}\Sigma\left(\frac{1}{a}+\frac{2}{b}\right)=\sqrt{3}\Sigma\frac{1}{a}=\sqrt{3}\)

Khách vãng lai đã xóa
Vinh Lê Thành
Xem chi tiết
Toi da tro lai va te hai...
31 tháng 5 2020 lúc 10:51

\(1-\frac{a^2b}{2+a^2b}\ge1-\frac{a^2b}{3.\sqrt[3]{a^2b}}\)\(\rightarrow1-3\sqrt[3]{a^4b^2}=3.\sqrt[3]{ab.ab.a^2}\rightarrow.....\)

Khách vãng lai đã xóa
Tran Le Khanh Linh
31 tháng 5 2020 lúc 10:53

BĐT cần chứng minh tương đương với \(\frac{a^2b}{2+a^2b}+\frac{b^2c}{2+b^2c}+\frac{c^2a}{2+c^2a}\le1\)

Áp dụng BĐT Cauchy ta có: \(2+a^2b=1+1+a^2b\ge3\sqrt[3]{a^2b}\)

Do đó ta được \(\frac{a^2b}{1+a^2b}\le\frac{a^2b}{3\sqrt[3]{a^2b}}=\frac{a\sqrt[3]{ab^2}}{3}\)

Hoàn toàn tương tự ta được \(\frac{a^2b}{2+a^2b}+\frac{b^2c}{2+b^2c}+\frac{c^2a}{2+c^2a}\le\frac{a\sqrt[3]{ab^2}+b\sqrt[3]{bc^2}+c\sqrt[3]{ca}}{3}\)

Cũng theo BĐT Cauchy ta được \(\sqrt[3]{ab^2}\le\frac{a+b+b}{3}=\frac{a+2b}{3}\)

\(\Rightarrow a\sqrt[3]{ab^2}\le\frac{a\left(a+2b\right)}{3}=\frac{a^2+2ab}{3}\)

Tương tự cũng được \(a\sqrt[3]{ab^2}+b\sqrt[3]{bc^2}+c\sqrt[3]{ca}\le\frac{\left(a+b+c\right)^2}{3}=3\)

Từ đó ta được\(\frac{a^2b}{2+a^2b}+\frac{b^2c}{2+b^2c}+\frac{c^2a}{2+c^2a}\le1\)

Vậy BĐT được chứng minh. Dấu "=" xảy ra <=> a=b=c=1

Khách vãng lai đã xóa
Lưu Thùy Dương
15 tháng 6 2020 lúc 17:36

1njfnjgjggnvfkgnbmvfvm 

Khách vãng lai đã xóa
Nguyễn Hải Minh
Xem chi tiết
Lê Tài Bảo Châu
9 tháng 8 2021 lúc 16:42

Ta có: \(2a+b^2=2a\left(a+b+c\right)+b^2=b^2+2a^2+2ab+2ac\)

\(\ge4ab+2ac+a^2\)

\(\Rightarrow\frac{a}{2a+b^2}\le\frac{a}{4ab+2ac+a^2}=\frac{1}{4b+2c+a}\)

\(\le\frac{1}{49}.\frac{49}{4b+2c+a}=\frac{1}{49}.\frac{\left(4+2+1\right)^2}{4b+2c+a}\)

\(\le\frac{1}{49}\left(\frac{16}{4b}+\frac{4}{2c}+\frac{1}{a}\right)=\frac{1}{49}\left(\frac{4}{b}+\frac{2}{c}+\frac{1}{a}\right)\)

CMTT: \(\frac{b}{2b+c^2}\le\frac{1}{49}\left(\frac{4}{c}+\frac{2}{a}+\frac{1}{b}\right);\frac{c}{2c+a^2}\le\frac{1}{49}\left(\frac{4}{a}+\frac{2}{b}+\frac{1}{c}\right)\)

\(\Rightarrow\frac{a}{2a+b^2}+\frac{b}{2b+c^2}+\frac{c}{2c+a^2}\le\frac{1}{7}\left(\frac{1}{a}+\frac{1}{b}+\frac{1}{c}\right)\)( đpcm )

Khách vãng lai đã xóa